infirmation de RH ?

Bonjour, sur l'arXiv aujourd'hui est paru une prépublication où l'auteur dit démontrer que RH est fausse. Le papier est ici (en postscript ou pdf au choix): http://arxiv.org/abs/math.NT/0703367

Recherche faite sur google, l'auteur semble être un ancien chercheur et vice-président d'université qui a aujourd'hui 77 ans et travaille sur ce sujet depuis une quinzaine d'années, voir https://lists.cs.columbia.edu/pipermail/ornet/2006-July/011372.html

Est-ce que les spécialistes du forum ont le temps de décortiquer la chose et nous dire ce qu'ils en pensent ? Il y a une vingtaine de pages avec des notations tout à fait usuelles on dirait, la stratégie générale étant un raisonnement par l'absurde: il suppose RH vraie et arrive à une contradiction.

Réponses

  • bonjour,
    sur ArXiv, on trouve beaucoup de choses .. amusantes, comme, par exemple, une célèbre démonstration du théorème de Fermat (enfin, je veux dire : célèbre .. sur ce forum..).
    à force, ça rend un peu méfiant.

    (mais si par extraordinaire, c'était sérieux, ça ferait beaucoup de peine à notre ami Sylvain, qui s'acharne depuis plusieurs années à vouloir démontrer RH..).
  • Ca a été posté par un mathématicien, c'est donc vraisemblablement non farfelu (ce qui ne dit rien sur l'exactitude du papier). On sera sans doute fixés rapidement...
  • Pour peu que cela puisse intéresser, je suis en train de regarder ce preprint...

    Premier point : je n'ai jamais entendu parler de cet auteur.

    Second point : les références historiques au début me semblent exactes, sauf en ce qui concerne la date la réfutation de la conjecture de Mertens par Odlyzko & Te Riele, qui est plutôt 1985/6 que 1983...

    Troisième point : c'est très pénible à lire (beaucoup de notations avec des primes, des tildes, etc), ce qui rend d'instinct suspect ce genre de papier.

    Borde.
  • Bonjour,

    J'adore à la page 18 la contradiction qui prouve que RH est fausse. Si ce type a réussi à montrer RH est fausse en arrivant au fait que sinon l'on obtiendrait $\displaystyle e^{\ln(x)}<x$ pour un certain $x$, ca serait très marrant tellement la condition est simple.

    sk.
  • Article d'autant plus suspect, car apparaissant juste en dessous d'un article de Connes:)

    Joaopa
  • Bah, suspect ou pas, il est juste ou faux et c'est cela qui compte.
  • Bonjour,

    Le lemme 7 me semble faux. la somme de gauche est au signe pres la derivee
    de zeta qui a un pole d'ordre 2 et non d'ordre 1 au point 1. L'erreur
    vient du fait que dans sa preuve l'auteur considère f'(s'), dérivée
    d'une fonction f définie avant qui est certes continue pour s'>1 (il
    utilise le terme "regular"), mais pour autant pas bornée au voisinage de
    1 comme il l'affirme. Or on retrouve ce lemme 7 invoqué page 13, et en
    particulier dans l'equation 17 qui est le point clé de sa preuve on dirait.

    Pour le reste ce qui precede ce lemme 7 m'a l'air globalement juste, mais
    comme dit borde c'est quand meme pas facile a lire....

    A+

    eric
  • Oups je corrige, ce n'est pas la derivée de zeta qui apparait dans le
    membre de gauche de l'égalité du lemme 7.
    Cela dit le fait que f' soit non borné au voisinage de 1 (page suivante)
    me semble incompatible avec la conclusion du lemme 7. Quelqu'un peut confirmer?

    A+

    eric
  • Je n'ai lu que les échanges du forum sur l'article en question. Mon expérience m'apprend que dans un raisonnement par l'absurde, on arrive souvent à des inégalités du type $0>0$, ce qui ne constitue pas une contradiction, car $0>0>0$. C'est tout ce que je peux en dire.
  • euh... comment definis tu '$<$' ?
  • En fait, ce n'est pas une question de définition. Il m'est très souvent arrivé, dans un raisonnement par l'absurde sur une proposition juste, d'aboutir à une inégalité du type $X>X$. Comme la proposition était vraie, j'ai appris à ne pas en tenir compte.
    Dans certaines démonstrations d'égalités du type $A=B$, je triche souvent en démontrant $A>B>A$. Les inégalités rigoureusement justes sont $A\geq{B}\geq{A}$.
    En fait, ça peut se comprendre comme suit :
    $A>0$ signifiant qua $A$ n'est pas négatif et $A<0$ que A n'est pas positif. Un nombre ni positif, ni négatif est nul. Ca vaut ce que ça vaut comme raisonnement, mais c'est très efficace.
    J'ai bien peur que cet article n'aboutit pas à une contradiction !
  • rien compris !
    (si quelqu'un pouvait revenir au sujet SVP..)
  • Salut Jonas.

    Ce que tu racontes est bizarre. Soit tu ne donnes pas le sens habituel à < (et c'est pour ça que Jobhertz te demande une définition de <), soit tu plaisantes.

    Cordialement
  • Veuillez me pardonner : je voulais donner une petite explication, mais ça crée du grabuge, je me retire de la discussion, continuez sans ma modeste intervention !
  • L'intérêt des grandes conjectures comme l'hypothèse de Riemann ne réside pas dans leur infirmation ou leur confirmation. Ce qu'on espère, c'est que la réponse passera par l'invention de nouvelles méthodes et une nouvelle compréhension des objets en question. Ainsi une infirmation ou confirmation de l'hypothèse de Riemann devrait, par exemples, nous permettre de comprendre le lien avec la conjecture de Weil (l'analogue pour les corps de fonctions) ainsi que pour toute une classe de fonctions L.

    Ainsi, la première preuve de Wiles du théorème de Fermat méritait d'être prise au sérieux (par opposition aux inombrables ``démonstrations'' précédentes) bien qu'elle se soit avérée éronée parce qu'elle introduisait de nouvelles méthodes de déformations de représentations galoisiennes et parce qu'elle inscrivait le théorème dans le cadre plus général du programme de Langlands.

    Les articles comme celui-ci qui prétendent démontrer une célèbre conjecture par une série de manipulations de niveau deug se peuvent pas être sérieux parce que même dans le cas improbable où la démonstration serait juste, on ne comprendrait pas mieux le problème ce qui la rendrait vide de sens.
  • Memento : si la preuve/infirmation de la conjecture semble ne pas apporter d'idées on sera déçu, c'est certain (et c'est un euphémisme). Mais ce n'est pas pour cela qu'une preuve/infirmation qui semble ne pas apporter d'idées est fausse...

    Disons plutôt que, vu la trogne de la preuve, on se surprend à avoir envie qu'elle soit fausse.
  • Pour une fois, je suis en désaccord avec mon ami Eric C...Mais ce qui me semble plus curieux, c'est que ce lemme 7 (tout au moins la dernière majoration) est connu pour être vrai {\it sans condition}, c'est-à-dire qu'il est vrai sans HR. En fait, on peut montrer que, pour tout $\sigma > 1$ réel, on a : $$\sum_{p} \frac {\ln p}{p^{\sigma}} < \frac {- \zeta'(\sigma)}{\zeta(\sigma)} < \frac {1}{\sigma - 1}.$$

    Référence : {\bf Hall & Tenenbaum}, {\it Divisors}, Cambridge Univ. Press (1987), lemma 70.1 page 146.


    En revanche, ce papier est vraiment très pénible à lire, ce qui ne plaide pas en sa faveur.

    Borde.
  • Il dit bien au début que certains lemmes sont indépendant de RH mais qu'il rajoute ça systématiquement pour mettre en exergue son raisonnement.

    Essayons de décortiquer dans l'ordre, quitte à changer ses notations si elles sont trop pénibles: le lemme 1 ça vous semble correct ou non ?

    Je reprends son raisonnement:

    énoncé du lemme 1:
    Soit l'intervalle $I_n:=(\frac{1}{2}+in ; \frac{1}{2}+i(n+1))$ sur la ligne critique. Alors il existe un point $\frac{1}{2}+iT$ et une constante A tels que pour tout zero de $\zeta$ de la forme $\frac{1}{2}+i\gamma$ dans $I_n$ on a $|T-\gamma|>\frac{A}{\log n}$. Cela donne ainsi une région sans zéros de $\zeta$ dans chaque $I_n$.

    sa preuve:
    Soit $N$ le nombre de zeros $z_1,\dots,z_N$ de $\zeta$ dans $I_n$. Alors il est connu que $N\leq A_1\log n$ où $A_1\geq 1$ est une certaine constante et $n\geq n_0$ pour un certain $n_0$.

    On va démontrer le lemme 1 par l'absurde: on suppose ce résultat est faux, c'est à dire qu'il n'existe pas de $T$ comme dans l'énoncé quelque soit la constante $A$.

    Mais alors les milieux des segments $(\frac{1}{2}+in;z_1),(z_1;z_2)\dots (z_N;\frac{1}{2}+i(n+1))$ sont à une distance des extémités de leur segment respectif qui est inférieure stricement à $\frac{A}{\log n}$ quel que soit $A$. Donc la longueur totale de $I_n$, qui est 1, est majorée comme dans son texte: $1\leq \frac{A}{\log n} \times 2 \times nb\ segments \leg \dots \leq 4 A A_1$ (les majorations sont effectivement valides).

    On vient d'obtenir que $A>\frac{1}{4A_1}$, donc en prenant $A=\frac{1}{4}{A_1}$ on obtient une contradiction ce qui prouve ce lemme 1.

    Je trouve le raisonnement valide à condition que l'on me donne une référence (ou une preuve) pour son "il est connu que $N\leq A_1\log n$" du début.
  • Salut Borde,

    Merci pour ta remarque. Le lemme 7 est donc juste.
    En attendant la preuve qu'il en donne (puisqu'il se donne la
    peine d'en donner une bien que ce soit un resultat apparemment connu)
    ne me parait pas valide en l'etat puisque à l'equation 9 il invoque le fait
    que f' est bornée comme une conséquence du lemme 4, or au lemme
    4 il montre que f est bornée mais pas nécessairement f' (meme si par ailleurs
    f' est peut-etre effectivement bornée sur $]1;+\infty[$,
    ca n'est pas justifié dans le texte).
    On a
    $$
    f' = - \sum_p \sum_{m=2}^\infty \log(p)/p^{ms}
    = - \sum_p \log(p) \frac{p^{-2s}}{(1-p^{-s})}
    $$
    Il est aussi surprenant qu'il justifie le lemme 6 qui est du niveau 1ere S....

    Je vais donc appronfondir car j'en étais resté la.

    A+

    eric
  • Oui, j'avais corrigé ce que tu me disais entre temps, à savoir sa preuve est aussi douteuse qu'inutile, puisque le résultat existe déjà (une application de la formule d'Euler-MacLaurin).

    Gedeon,

    j'avais bien lu l'en-tête que tu cites, mais je trouve assez inutile de mettre des HR (ou RH en anglais) partout, même (et surtout) lorsqu'il n'y en a pas besoin. Du coup, comme sa "preuve" est une démonstration par l'absurde, je me suis demandé ce que cela donnait en considérant ce lemme 7 vrai {\it même sans avoir HR}, alors que l'auteur semble penser qu'il n'est valable que sous HR, ce qui est faux (je parle de l'inégalité, bien sûr, qu'il utilise dans sa preuve).

    Quant au nombre de zéros, le résultat est classique (conjecturé par Riemann, puis démontré par Von Mangoldt à la fin du 19ème), voir n'importe quel livre de théorie analytique des nombres : par exemple, {\bf Tenenbaum}, {\it Introduction à la théorie analytique et probabiliste des nombres}, SMF (1995), théorème 12 page 155.

    Borde.
  • Un premier bilan après avoir parcouru les 10 premières pages :

    1. {\bf page 6, ligne 5}. Je ne comprends pas où est passé le $\exp \left ( (\log A) \widehat {A} \widehat {\Omega}(t) \right )$ ?

    2. {\bf page 7, lemme 6}. Je trouve la preuve bien compliquée, là où l'inégalité classique $|\sin x| \leqslant |x|$ suffisait.

    Borde.
  • Salut Borde,

    Pour le 1 je pense qu'entre les 2 lignes il redimensionne la constante A
    (qui n'est donc pas le meme sur les 2 lignes) afin que l'inegalitée
    soit vraie pour t assez grand ce qui est sont but, et en utilisant le
    fait que pour t assez grand on a log(log(log(t))) plus grand que n'importe
    quelle constante choisie au depart.

    En tous cas c'est comme ca que je le comprends..

    A+

    eric
  • L'auteur a, semble-t-il, corrigé des erreurs et mis une nouvelle version en ligne :
    \lien{http://arxiv.org/abs/math.NT/0703367}
  • Je viens de pousser un peu plus loin la lecture de ce papier
    est c'est de plus en plus confus...
    L'equation 15 pose un probleme de formulation car il defini $t=t_0+\delta$
    avec $\delta = A/\log(n)$ puis $\delta = min |t-\gamma|$ ou $\gamma$
    decrit les zeros de zeta (sur la droite critique)... Quelle definition est
    la bonne? Probablement la 1ere mais le role de (15) n'est pas clair...

    Ensuite page 12 on a un B qui disparait comme par enchantement...
    Enfin la plupart des majorations sont tellement grossieres que
    pour obtenir une contradiction du type $0<0$ il faut vraiment
    avoir laissé tomber un terme essentiel quelque part...

    Pas très convaincant tout ca.

    Eric
  • Salut Eric,

    Quant à moi, j'ai laissé tomber ce papier, car je dois faire le "referee" d'un article (autrement plus sérieux, celui-là) pour le Journal of Integer Sequences, ce qui me prend pas mal de temps.

    J'adhère avec ta dernière remarque, il ne me semble pas possible de réfuter HR par un si simple argument.

    Borde.
  • Quant à moi il ne me semble pas possible de réfuter HR par quelque argument que ce soit...
    Comment ça je suis de mauvaise foi ?:D
  • Sans vouloir vous casser je trouve cela dommage que pour des raisons de forme (langue, degré de détail, complexité des notations) des gens qui ont autant de connaissances que vous ne se prennent pas une semain entière pour véfifier tout cela? Quel gâchis! Après tout c'est quand même une des conjectures les plus importantes des mathématiques!

    Bon j'ai compris en lisant que vous faisiez des efforts pour la comprendre mais nous les novices niveau bac+1,2 ou 3 on aimerait bien savoir au moins si c'est juste ou faux sa démo!!!

    C'est pas de notre faute si on les a pas encore ces connaissances!!!

    Mais je vous jure que si je les avais...je passerais des journées dessus sans problème!!! Des semaines, un mois s'il le faut, je ne prendrais pas de vacances!
  • SXB : si les matheux passaient leur temps à lire les tentatives de preuves de conjectures célèbres, ils ne feraient plus de math...

    L'auteur du papier en question l'a certainement soumis à une revue et un ou deux gus sont du coup certainement en train de vérifier le papier (ou ont déjà fini). Il n'est pas utile (et pas souhaitable) que les 100.000 matheux professionnels cessent toute activité pour lire le papier...
  • SXB a écrit:
    C'est pas de notre faute si on les a pas encore ces connaissances!!!
    Mais je vous jure que si je les avais...je passerais des journées dessus sans problème!!! Des semaines, un mois s'il le faut, je ne prendrais pas de vacances!

    J'adore. Parce que tu crois que la connaissance ça tombe du ciel ? Comment crois-tu que les gens compétents dans ce domaine (je n'en fais pas partie) ont acquis cette compétence ? Justement en lisant des articles, et en faisant des recherches (manuels, livres de recherches, autres articles) sur chaque concept qu'ils ne comprennaient pas lorsqu'ils tombaient sur un os.

    Ne te gène donc surtout pas pour t'y mettre si cet article te passionne : lis-le, passe-y ton mois (ou ton année). En plus, tu a la chance de connaitre un forum génial où tu pourras poser toutes les questions nécessaires lorsque tu tomberas sur quelque chose que tu ne comprends pas, avec l'assurance, au minimum , d'obtenir de bonnes références.

    PS1 : Mon ton semble peut-être arrogant. Si c'est le cas je m'en excuse sincèrement. Ce n'est pas mon intention. Je pense vraiment que tu devrais essayer de lire cet article puisque le sujet t'intéresse, et que rien ne t'en empêche (sauf peut-être la forme : langue, degré de détail, complexité des notations)

    PS2 : Désolé si j'ai nourri (ou pire, si je déclanche) un troll. J'ai eu un mois difficile.
  • Désolé pour toi que ce mois fut difficile, Barbu, j'espère que les jours à venir te seront plus favorables (en tout cas, je te le souhaite...).


    Je vais répondre également à SXB, car il me semble que son propos me concerne également (toutefois, si cela n'était pas le cas, qu'il ne tienne pas compte de ce que je vais dire...) :

    Dans la communauté des spécialistes de HR, il y a (à peu près) convergence de vue sur sa véracité. En fait, il existe de nombreux indices pour croire à cette véracité : tout d'abord, il y a le théorème de Bombieri-Vinogradov (1965) qui constitue un vrai cap en sa faveur, puisque l'on peut le considérer comme "une preuve" de HR "en moyenne". Il y a aussi le résultat (profond) d'André Weil (1948) qui stipule de HR est vraie pour les corps de fonctions. N'oublions pas non plus que l'on sait aujourd'hui que plus de 40 \% des zéros de $\zeta$ sont situés sur la droite critique.

    Bref, pas de preuve, certes, mais un faisceau de présomptions qui font penser à la véracité de HR, et, surtout, on ne comprend(rait) pas pourquoi elle serait fausse (j'entends par là : on ne trouve pas de raison arithmétique profonde qui viendrait infirmer HR).

    Aussi, lorsque quelqu'un (inconnu pour moi), et ce, en deux coups de cuillère à pot, prétend réfuter HR, on ne peut pas dire que son papier arrive dans les meilleures conditions, surtout lorsque la démonstration est rédigée avec un abus évident de notations souvent superflues, ce qui rend suspect ledit papier, alors que celui-ci aurait dû, vu la difficulté du sujet, être d'une clarté sans faille. Il y a eu par le passé de telles tentatives (un auteur en a même fait un livre. Faire une recherche sur le forum...) !!

    Ceci dit, tout un chacun peut télécharger ce preprint et (essayer de) se faire sa propre opinion dessus. Il ne me paraît pas nécessaire de connaître sur le bout des doigts la théorie, et, au besoin, Eric C., moi ou d'autres peuvent éventuellement intervenir pour tel ou tel point précis (voir nos mesages respectifs plus haut).

    Pour ma part, je conseillerais plutôt l'article suivant, plus sérieux à mon sens :

    \lien {http://fr.arxiv.org/PS_cache/math/pdf/0703/0703570.pdf}

    Borde.
  • Message pour Jonas :
    Tes trucs où tu arrives sur des x<x ou a<b<a pour prouver a=b (tu vois de quoi je parle) m'intriguent. J'aimerais bien que tu développe et que tu donnes des exemples. Ayant tjs moi même pratiqué des millions de trucs louches qui ont tjs marché car je connais les limites de mes trucs louches. Donc voilà si tu pouvais développer tes idées je t'en saurais gré. A la limite commence un autre sujet avec un titre explicite pour que je le retrouve.
  • Merci à tous pour vos réponses, surtout à Borde qui m'a apporté tant de renseignements. ceci dit, il est vrai qu'à un noveau en dessous de bac + 5 il est tellement facile de vérifier les démos en comparaison avec ce genre de démos là qu'il est en effet beaucoup plus important d'être clair.

    Je me suis un peu emporté dans ces propos, désolé, je suppose bien qu'il doit y avoir tant de preuves erronnées.

    Mais quel culot de présenter une démo sans la vérifier!!!! Même si je devais y passer dix ans, je ne publierais pas une démo avant d'en être certain!

    J'ai déjà tourvé quelques jolies petites démos peut-être inédites (car originales) pour certaines mais qui ne font pas appel à des notions bien grandioses, qui ne font pas appel à beaucoup d'autres résultats et qui à priori ne donnent rien de révolutionnaire même si c'est probablement inédit.

    Mais on n'emploie pas dans une publication un résultat que l'on ne connait pas.

    Comme certains l'ont si bien dit, on ne peut pas tricher en maths, alors pourquoi faire perdre du temps aux autres?
  • Salut SXB,

    Pour ma part, je ne pense pas que tu te sois emporté, et, quand bien même, on a encore le droit de s'emporter... le tout est, finalement, d'arriver à obtenir les renseignements souhaités.

    De plus, je répète qu'il est fort probable que les gens "qui s'y connaissent" restent à la disposition des questions éventuelles qui pourraient être posées. En ce qui me concerne, même si, actuellement, je n'ai plus le temps (ni l'envie, il faut bien le dire) de relire ce papier, je reste à la disposition de quiconque pour des questions ponctuelles, et ce, dans la limite de mes connaissances.

    Borde.
  • Mais au juste, quelles seraient les conséquences si par malheur RH (je persiste à utiliser l'abréviation anglo-saxonne, HR restant pour moi synonyme d'Hypothèse de Récurrence, de même que MST ne peut guère signifier que...Milieu Semi Transparent !(:P)) se révélait fausse ?
  • Pour toi, un banissement à vie du forum serait le minimum, non.. ?? ;)
  • Sylvain,

    RH peut aussi être le facteur rhésus...

    Plus sérieusement, la réfutation de HR poserait un certains nombres de problèmes, au moins quant à notre compréhension des phénomènes arithmétiques.

    Tout d'abord, et c'est pour moi l'un des points principaux, il faudrait que l'on m'explique pourquoi Bombieri-Vinogradov est vrai alors qu'HR est fausse. Il n'y a pas à proprement parlé de contradiction réelle entre ces deux résultats, mais cela me gênerait vraiment !...

    De plus, HR faisant tomber un certain nombre d'autres conjectures (Lindelöf, par exemple), il faudrait alors s'attaquer indépendamment à ces conjectures, sans espérer le secours de HR. Or, pour prendre comme exemple l'hypothèse de Lindelöf, {\it on en est encore loin} avec les outils actuels (à tel point que je me demande si les gens n'attendent tout simplement pas HR pour en déduire les conjectures plus faibles !...).

    Borde.
  • bonjour à tous
    si je comprends bien le résumé de Borde ,
    si RH était fausse cela napporterait rien de plus;

    on pourrait toujours s'en servir jusqu'à une certaine limite qui reste à définir
    et au moins jusqu'à cette limite Rh est vraie!

    d'où, jusqu'à cette limite qui reste inconnu elle est vraie et quand bien même elle serait fausse à l'inconnu, cela n'enlève rien à tous les travaux actuel.
    et pour certaine conjecture il faut attendre d'avoir des moyens suffisants , ou la lumière qui viendra la confirmer:P
  • {\it si je comprends bien le résumé de Borde ,
    si RH était fausse cela napporterait rien de plus}

    Je n'ai pas exactement dit cela, toute information quant à la véracité d'une telle hypothèse, vraie ou fausse, serait certainement riche de renseignement. Par exemple, le premier zéro éventuel de $\zeta$ qui {\bf ne serait pas} sur la droite critique serait certainement une constante arithmétique d'une grande valeur...

    Mais...

    Voici néanmoins quelques arguments qui plaident en faveur de HR. {\bf Ce ne sont pas} des preuves, mais peuvent aider tout un chacun à se faire une opinion :

    1. On sait que presque tous les zéros de $\zeta$ sont très proches de la droite critique. Plus précisément, plus de 99\% des zéros $\rho = \beta + i \gamma$ (notation officielle des zéros de $\zeta$) vérifient $\displaystyle {\left | \beta - \frac {1}{2} \right | \leqslant \frac {8}{\ln |\gamma|}}$.

    2. HR implique aussi que tous les zéros des dérivées de la fonction $\xi(s) = \frac {s(s-1)}{2} \pi^{-s/2} \Gamma (s/2) \zeta(s)$ sont aussi sur la droite critique, et on a montré que plus de 99 \% des zéros de $\xi'''(s)$ sont sur la droite critique. Il semblerait que Norman Levinson, peu de temps avant sa mort, pensait avoir trouvé une méthode (utilisant une espèce de "théorème de Rolle inversé") qui lui aurait permis de passer d'une proportion connue de zéros de $\xi'(s)$ sur la droite critique à une proportion pour $\xi(s)$ sur cette droite, et de même pour passer des zéros de $\xi''(s)$ à ceux de $\xi'(s)$, etc. Mais personne n'a pu depuis faire fonctionner ce principe. Rappelons tout de même que ce chercheur a inventé une technique pour compter la proportion de zéros sur la droite critique (voir son article de 1974).

    3. La symétrie des nombres premiers : si HR était fausse, il y aurait d'étranges irrégularités dans la distribution des nombres premiers, tellement étranges que personne n'y croit vraiment.

    Borde.
  • bonjour Mr Borde

    excusez moi de vous embêter encore un tout petit peu,
    mais cette phrase me fait vous poser cette question complémentaire :

    *******************************************************************************
    3. La symétrie des nombres premiers : si HR était fausse, il y aurait {\bf d'étranges irrégularités dans la distribution des nombres premiers}, tellement étranges que personne n'y croit vraiment.
    ********************************************************************************
    de quelle nature serait l'irégularité dans la distribution par exemple ?

    personnellement je pensais que la distribution des nombres premiers est, d'une façon naïve, de boucher les espaces vides entre leurs produits, c'est à dire les nombres composés produits par les premiers, afin de respecter l'écart de 1 entre chaque entier naturel.

    De ce fait ce sont les composés qui sont distribués par les nombres premiers et qui donc, leur assignent une place.

    Un exemple, je suprime les mutliples de 2, 3 et 5
    Je suis dans l'ensemble des entiers congru p(30) soit 26,666...\% des entiers naturels. qui commence par 1, 7 , 11 , 13 ,17 , 19, 23 , 29 on peut effectivement remplacer 1 par 31,

    Le premier composé sera alors 49, puis 77 entre ces 2 nombres il ne peut y avoir que des premiers p(30)

    On remarque alors que les premiers sont toujours en avant de leur produit.
    Pourquoi parle-t-on {\bf de distribution} des nombres premiers ?
    Est-ce que l'irrégularité sous-entend que l'écart de 1 entre chaque entier naturel ne serait plus respecté par exemple,
    ou que le nombre de premiers entre deux espaces ne serrait oscillatoire c'est à dire :
    De A à B on a x premiers, puis de B à C : u premiers avec u < x et de C à D : v premiers avec v > u par exemple ?

    On peut aussi envisager qu'à une distance très lointaine dans un espace de 100 entiers naturels on retrouve la même proportion de premiers comme pour les 22 premiers de 1 à 100 en enlevant 2,3 et 5... Non ?

    [En LaTeX, il faut banaliser le \% : \verb*=\%=. AD]
  • Je pense que Borde évoque la fonction $\pi(x)$ (le nombre de nombres premiers inférieurs à $x$). En fait RH équivaut à:
    $\exists C>0,\ \vert\pi(x)-Li(x)\vert <C\sqrt(x)\ln x$

    Cette inégalité traduit une certaine "régularité" de la fonction $\pi(x)$, et donc de la distribution des premiers parmi les entiers naturels.
    Le principal intéressé confirmera ou non ces propos le cas échéant.
  • C'est cela (je n'avais pas vu la remarque de l.g).

    Borde.
  • Bonjour Sylvain , Borde..

    Merci pour ce compliment,

    En fait comme la distribution des premiers, est forcément régulière, même si les entiers naturels composés occupent de plus en plus de place, il y aura toujours des espaces avec plus ou moins de premiers, donc une courbe oscillatoire par rapport à zéro.

    En supposant RH vraie, confirmerait-on l'infinité de premiers jumeaux, ou l'inverse, c'est à dire que RH serait vrai. Car je suppose qu'une distribution régulière de premiers, implique un écart minimum de 2 entre deux premiers, comme 4,6,8... etc
    Est-ce que cette formule permet de calculer le nombre de nombres premiers entre deux points, je veux dire par là entre A et B puis entre B et C ?
    Ou en calculant les différences on constate plus ou moins de premiers.
    Si oui, est-ce que l'on remarque un nombre de premiers plus ou moins important en fonction de ces points ? Voici un exemple :

    Prenons la famille des premiers 7(30) de 0 à 90mds puis de 0 à 180mds ... 0 à 450

    F7 : ¡ú 89999999797 = 465 309 093 nP
    F7 : ¡ú 179999999707 = 904 621 181 nP
    F7 : ¡ú 269999999857 = 1 335 134 285nP
    F7 : ¡ú 359999999827 = 1 760 099 445nP
    F7 : ¡ú 449999999767 = 2 181 058 528 nP
    on remarque que le nombre de premiers 7(30) pour 90 mds de plus, chute de moins en moins ce qui fait remarquer qu'à une certaine distance, on se retrouvera pour 90 mds de plus avec un nombre de premiers supérieur à la dernières série précédente.
    Il en est de même pour les 8 familles.

    Mais ce qui est le plus intérressant dans la famille F23(30) où l'on peut supposer qu'il y aurait moins de premiers par rapport à F7(30) du fait que cette F23 comporte les deux couples de jumeaux f11 et F13 ainsi que F17 et F19 donc en supposant un nombre fini de jumeaux, il y aurait une irrégularité voire un paradoxe c'est à dire moins de premiers 23(30) et plus de premier 7(30) ce qui est faux. De temps en temps plus de 23(30) et d'autre fois le contraire.

    Le nombre de premiers par famille reste équivalent la courbe du nombre de premiers, est oscillatoire lorsque n tend vers l'infini.

    S23 : ¡ú 89999999993 = 465308 258 nP
    S23 : ¡ú 179999999873 = 904 628 614 nP
    S23 : ¡ú 269999999903 = 1 335 129 326 nP
    S23 : ¡ú 359999999753 = 1 760 101 423 nP
    S23 : ¡ú 449999999963 = 2 181 059 655 nP

    Peut-on avec cette formule calculer par Famille (on peut faire la moyenne bien sûr) ?


    [Je n'ai pas su à quoi correspondent les codes ¡ú et nP :-( AD]
  • Je n'ai absolument rien compris.
  • bonlour et excusez moi.
    les codes; jusqu'à et nP = nombre premier ex:

    Série 23(30): nombr de prem de 0 à 449 999 999 963 = 2 181 059 655 nP .


    ma question sylvain
    si Rh est vrai, la formule que tu donnes permet de constater une certaine régularité dans le calcul du nommbre de premier.
    je suppose que cette formule permet aussi de calculer le nombre de premiers.

    1)
    mais est ce que cela permet de donner le nombre de premier (estimation) entre deux points exemple entre 1000 et 10 000 puis entre 10 000 et 20 000.
    2)
    ou, peut on constater si Rh et vrai que la courbe du nombre de premiers lorsque N tend vers l'infini, que ce nombre de premiers est oscillatoire.
    c'est à dire qu'entre deux points, il peut y avoir plus de premiers qu'entre deux points précédents.
    car je suppose que tous ces zéros qui sont sur la bande critique sont en rapport avec la distribution du nombre de premiers, ne connaissant pas exactement le role de cette hypothése de R , j' en ai compris que ces zéro sont comme des points, avec plus ou moins de premiers, mais sans pour autant dépasser le nombre réel de premiers donné par la fonction pi(x).

    le fait de considérer un nombre de premiers oscillatoire :
    plus, moins , plus, moins, permet de supposer un peu plus, que le nombre de premiers jumeaux est infini. De plus, je pense que si ces premiers jumeaux était en nombre finis, il y aurait une irégularité dans le nombre de premiers.

    maintenant, si cette formule à pour but uniquement de donner une estimation de premier par rapport à X sans autre précision...j'espère qu'elle a plus d'utilité pour d'autre fonction dans ce cas .
  • J'ai déjà parlé ici-même de la différence $\pi(x+y) - \pi(x)$ donnant le nombre de nombres premiers dans l'intervalle $]x,x+y]$.

    Il faut bien comprendre que ce problème est d'une nature plus difficile que le TNP, et tiens en haleine bon nombre de chercheurs depuis bon nombre d'années.

    Quels sont les résultats ?

    1. Avec des raffinements dernier cri de la méthode du grand crible, Montgomery & Vaughan (1973) ont démontré que : $$\pi(x+y) - \pi(x) < \frac {2y}{\ln y}$$ est valable pour tous réels $x,y \geqslant 2$, ce qui est un résultat remarquable. Notons aussi que les auteurs l'ont généralisé aux différences $\pi(x+y;q,a) - \pi(x;,q,a)$.

    2. On {\bf conjecture} que la borne inférieure est de même nature, mais, à l'heure actuelle, le problème est toujours ouvert. En 1992, Lou & Yao ont obtenu : $$\frac {0,969y}{\ln x} < \pi(x) - \pi(x-y) < \frac {1,031y}{\ln x}$$ valable pour $y = x^{6/11 + \varepsilon}$ et tout réel $x \geqslant x_0$ suffisamment grand. Par la suite, l'exposant $6/11$ a été amélioré par Baker & Harman et l'ont ramené à $0,525$.

    3. En 1981, Heath-Brown a obtenu : $$\pi(x) - \pi(x-y) = \frac {y}{\ln x} + O\left ( \frac {y}{(\ln x)^{45/44}} \right ),$$ valable "uniquement" si $x^{7/12} \leqslant y \leqslant x$.

    4. L'inégalité $\pi(x+y) - \pi(x) \leqslant \pi(y)$ ($x,y \geqslant 2$) reste encore et toujours une conjecture. En 1973, Montgomery et Vaughan ont obtenu : $$\pi(m+n) - \pi(m) < 2 \pi(n),$$ valable pour tous entiers $m \geqslant 1$ et $n \geqslant 2$.

    Voilà les principaux résultats connus en la matière actuellement.

    Borde.
  • merci Mr borde pour ces renseignements qui me permettent de comprendre un peu mieux la raison de cette Hypothèse de R.

    mais je trouve dommage que l'on s'en tienne à l'ensemble des entiers naturels

    car je pense que l'on aurrait peut être une meilleur estimation dans les entiers P(30) ou du moins une meilleur vision de leur répartiton, déja en ne travaillant que sur une des 8 familles;
    on risque de s'apercevoir que le résultat serait le même, que de travailler dans l'ensemble des entiers naturels et de là, reconsiderer leur répartiton en commençant par les premiers jumeaux puis les nombres de Fermat on pourrait bien avoir la surprise d'en trouver d'autre.

    les multiples de 2,3 et 5; n'apporte rien à la distribution des nombres premiers.

    mais peut être que ma supposition aussi n'apporterait rien de plus.

    la raison qui me fait dire cela, c'est que: plus l'ensemble est réduit meilleur est l'estimation.
Connectez-vous ou Inscrivez-vous pour répondre.